BDMO Primes!

p p is a prime number such that there exist positive integers n n and m m that satisfy the following equation:

p n + 144 = m 2 p^n+144=m^2

What is the sum of all possible values of m m ?


This problem appeared at the BDMO-2007 regionals.


This problem is from the set "Olympiads and Contests Around the World -1". You can see the rest of the problems here .


The answer is 48.

This section requires Javascript.
You are seeing this because something didn't load right. We suggest you, (a) try refreshing the page, (b) enabling javascript if it is disabled on your browser and, finally, (c) loading the non-javascript version of this page . We're sorry about the hassle.

4 solutions

Jubayer Nirjhor
May 23, 2014

Rewriting gives: p n = ( m + 12 ) ( m 12 ) p^n=(m+12)(m-12) . Let m + 12 = p i m+12=p^i and m 12 = p j m-12=p^j and so i + j = n i+j=n and i > j i>j .

Then we have:

p j ( p i j 1 ) = p i p j = ( m + 12 ) ( m 12 ) = 24 = 5 0 ( 5 2 1 ) = 2 3 ( 2 2 1 ) = 3 1 ( 3 2 1 ) . \begin{aligned} p^j(p^{i-j}-1) &=p^i-p^j \\ &=(m+12)-(m-12)\\ &=24=5^0(5^2-1) =2^3(2^2-1)=3^1(3^2-1).\\ \end{aligned}

From here we have 3 3 solutions: m = 13 , 15 , 20 m=13,15,20 with sum 48 \fbox{48} .

More paragraphs would have made your solution a little bit easier on the eye :)

Mursalin Habib - 7 years ago

Log in to reply

Awesome problem! I wish I can make problems as nice as the professional problem writers :P

Daniel Liu - 7 years ago

Log in to reply

Haha yeah I try, I try. It's very inspiring to see this level of awesome problem. :D

Finn Hulse - 7 years ago

pls tell the solution

Aman Real - 7 years ago

its really nice question

Kirti Srivastava - 7 years ago

I got the 13 and the 15 because 3^4 + 144 = 15^2 or 71 + 144 = 225 and 5^2 + 144 = 13^2 or 25 + 144 = 169 and 13 and 15 are primes. but 16^2 + 144 = 20^2 or 256 + 144 = 400 and 16 is not a prime. Then why 20? Kindly correct me if I am wrong.

Oliver Daniel - 7 years ago

Sorry. I made a mistake in my previous comment. 3^4 + 144 is 81 + 144 = 225 which is 15^2.

Oliver Daniel - 7 years ago

Great! Thanks!

Noel Lo - 5 years, 12 months ago

Same ;):) Way Too easy

Kaustubh Miglani - 5 years, 1 month ago

Did the same.

Shishir Shahi - 3 years, 7 months ago

You must write elaborated solution.

Mayank Chaturvedi - 7 years ago

Log in to reply

See the solution that @Rohan Arora has posted.

Jubayer Nirjhor - 7 years ago

Hit and trial goes here...

I did it that way. You can put the value of m starting from 13 till 20 and then you will find that 13, 15, 20 satisfy the equation and give us the value of p^ n as 5^2, 3^4, 2^8..

This looks like a series as p is decreasing in the form from 5, 3, 2 and n in the form of 2, 4, 8

As we don't have a prime number less than 2, our series comes to an end. So, the answer is 48(we have to assume that this is a series or a trend coming on).

Kartik Sharma - 7 years ago

Log in to reply

Absolute hit method of hit and trial. I appreciate your solving skill. Although your method was hit and trial, it requires reasnoning. Very nice.

Mayank Chaturvedi - 7 years ago

Log in to reply

@Mayank Chaturvedi Thanks! BTW, always remember ne'er to underestimate the power of hit and trial....;)

Kartik Sharma - 7 years ago
Rohan Arora
May 25, 2014

Okay here we go

Rearranging the equation and simple factoring gives us

p n + 144 = m 2 p n = m 2 144 p n = ( m 12 ) ( m + 12 ) { p }^{ n }+144={ m }^{ 2 }\\ { p }^{ n }={ m }^{ 2 }-144\\ { p }^{ n }=(m-12)(m+12)\\ \\

Also, p n p^n can be rewritten as p x ( p n x ) { p }^{ x }({ p }^{ n-x })

So,

p x ( p n x ) = ( m 12 ) ( m + 12 ) { p }^{ x }({ p }^{ n-x }) = (m-12)(m+12)

Therefore, assuming that p x < ( p n x ) p^x < ({ p }^{ n-x }) ,

p x = ( m 12 ) p^x=(m-12)

p n x = ( m + 12 ) { p }^{ n-x }=(m+12)

We can now say that

p x + 24 = p n x p^x+24=p^{n-x}

So the difference between two powers of p p is 24 24 . Now let us start testing with prime numbers, starting from 2:

2 0 + 24 2 y 2 1 + 24 2 y 2 2 + 24 2 y 2 3 + 24 = 2 y , a n d y = 5 { 2 }^{ 0 }+24\neq { 2 }^{ y }\\ { 2 }^{ 1 }+24\neq { 2 }^{ y }\\ { 2 }^{ 2 }+24\neq { 2 }^{ y }\\ { 2 }^{ 3 }+24={ 2 }^{ y },\quad and\quad y=5

We can see that further adding 24 to further powers of too will not result in a power of two as once you get to 2 5 2^5 , the difference between two consecutive powers of two is greater that 24.

Now we try with the next prime: 3

3 0 + 24 3 y 3 1 + 24 = 3 y , w h e r e y = 3 3 2 + 24 3 y { 3 }^{ 0 }+24\neq { 3 }^{ y }\\ 3^{ 1 }+24=3^{ y },\quad where\quad y=3\\ { 3 }^{ 2 }+24\neq { 3 }^{ y }

Once again we can see that continuing would be futile as the difference between 3 3 3^3 and 3 4 3^4 is greater than 24 24

Okay moving on to 5

5 0 + 24 = 5 y , a n d y = 2 5 1 + 24 5 y { 5 }^{ 0 }+24=5^{ y },\quad and\quad y=2\\ 5^{ 1 }+24\neq 5^{ y }

For the same reason as before, we can see that continuing with powers of 5 would be futile

Also, we cannot continue with any prime numbers after 5, because, as you can see, as the primes get bigger, the value of n n gets smaller. However, the smallest value of n n has already been reached (0). So continuing with 7 will not give us any solutions.

So now we can see that p n p^n must be :

5 0 , 3 1 , o r 2 3 { 5 }^{ 0 },{ 3 }^{ 1 },\quad or\quad { 2 }^{ 3 }

And:

5 0 + 12 = 13 3 1 + 12 = 15 2 3 + 12 = 20 { 5 }^{ 0 }+12=13\\ 3^{ 1 }+12=15\\ { 2 }^{ 3 }+12=20

Adding these three values of m m , we get:

15 + 13 + 20 = 48 15+13+20 = \boxed{48}

Really nice solution.

Shashank Rammoorthy - 6 years, 10 months ago

In line 13 one may observe that p^x has to divide 24.

However 24 = 2^3 * 3 * 1 or 24 p x + 1 = p n \frac{24}{p^x} + 1 = p^n for some n N n \in \mathbb{N} so this proof can be made even shorter with less cases to check.

Meer Kat - 5 years, 7 months ago

Rearranging to p n = m 2 144 = ( m + 12 ) ( m 12 ) p^n = m^2 -144 = (m+12)(m-12)

In order for LHS to be a prime power, both ( m + 12 ) (m+12) and ( m 12 ) (m-12) must be, with ( m 12 m + 12 ) (m-12 \mid m+12) .

m + 12 m 12 = 1 + 24 m 12 \frac{m+12}{m-12} = 1 + \frac{24}{m-12}

Therefore ( m 12 ) 24 (m-12) | 24 .

This gives m = 13 , 14 , 15 , 16 , 18 , 20 , 24 , 36 m = 13, 14, 15, 16, 18, 20, 24, 36 .

Although not the most efficient way, there are few enough cases to test one by one.

This is only valid when m = 13 , 15 , 20 : S u m = 48 m = 13, 15, 20: Sum = \boxed{48}

Nice solution indeed :)

Mahtab Hossain - 5 years, 11 months ago
Sayan Kumar
May 25, 2014

I have tried to give a simplified solution: One may think at the initial that there is huge set of numbers so sum can be anything. But it’s not like that. p^n=(m+12)(m-12)=p^i*p^j (say) So, p^i-p^j=(m+12)-(m-12)=24. ( Just think 5^3-5^2=100. And so p,i,j can’t be huge numbers.)
So thinking with the initial prime nos 2, 3 and 5 and taking m>12 (As p^n is positive) we get- ->m=13 for which p=5 ,i=2,j=0 ->m=15 for which p=3,i=3, j=1 ->m=20 for which p=2,i=5,j=3 So sum of all possible m’s are=13+15+20=48

I made a mistake. I forgot that 2 is a prime and that 2^8 = 256.

Oliver Daniel - 7 years ago

0 pending reports

×

Problem Loading...

Note Loading...

Set Loading...